LSAT and Law School Admissions Forum

Get expert LSAT preparation and law school admissions advice from PowerScore Test Preparation.

 bella243
  • Posts: 65
  • Joined: Apr 29, 2020
|
#75506
Adam Tyson says there author made a Mistaken Negation, while KelseyWoods says the author made a Mistaken Reversal. So which one is it?

My take is that there's a Mistaken Negation. Police Well paid-> Good legal system
Mist Negation: Police Well paid -> Good legal system
 Luke Haqq
PowerScore Staff
  • PowerScore Staff
  • Posts: 742
  • Joined: Apr 26, 2012
|
#75935
Hi bella243!

To your question, it's equally valid to refer to this as a Mistaken Negation or instead as a Mistaken Reversal, depending on how one initially diagrams it. Whether identified as a Mistaken Negation or a Mistaken Reversal, both demarcations identify a flaw in the reasoning given in the stimulus.

In this particular question, the author moves from the assertion, "there cannot be a good legal system where the police are not well paid," and concludes that "it follows that where the police are well paid there will be a good legal system." In other words, it starts with the following reasoning:
good legal system :arrow: police well paid
Contrapositive: police well paid :arrow: good legal system
Another way of saying this is that if one has a good legal system, then one knows that the police are well paid (or, if police are not well paid, then one cannot have a good legal system). From this alone, there's nothing guaranteeing that a well paid police force alone will produce a good legal system (e.g., this would also require competent lawyers and judges, access to courts, etc.). A good legal system implies well-paid police, but not vice versa. However, this mistake is precisely what the author of the stimulus concludes in saying that "it follows that where the police are well paid there will be a good legal system":
police well paid :arrow: good legal system
In reaching this conclusion, the author makes a flaw in reasoning. From the perspective of my original diagram, this flaw would be referred to as a Mistaken Reversal (because it reverses the variables). From the perspective of the contrapositive, however, it would be a Mistaken Negation (because the variables are in the right spots but aren't negated when they should be). Whether one identifies this as a Mistaken Negation or a Mistaken Reversal will depend on how one diagrams the sentence "there cannot be a good legal system where the police are not well paid"--my contrapositive above, for example, might be how others wrote out the initial diagram, but they both express the same thing.
 ahorita
  • Posts: 3
  • Joined: Aug 03, 2020
|
#79333
So, I had no problem diagramming the premise and the conclusion.
I even knew that it was mistaken negation(or can be called mistaken reversal in terms of contrapositive) situation.
I guess my problem was interpreting what the Q.stem meant by 'fails to establish that'

I thought, since the premise goes g.l.s --> p.w.p
and the theorist ends up with p.w.p --> g.l.s

what I need to look for as him 'failing to establish' is that / g.l.s --> p.w.p to strenthen p.w.p as a ncecessary condition that's irrelevant to the existence of sufficient condition. That's why I was so confused when I diagrammed through all the answer choices, since I couldn't find any that matches my prephrase.

I diagrammed D correctly w.p--> gls, and I know that it matches with the theorist's conclusion. I still don't understand why the theorist 'fails to establish' it. I thought generally flaw questions mandate us to take the premise and attack the conclusion. This seems like taking the conclusion granted and attacking the premise for not being able to support the conclusion.

Hope I made myself clear.
User avatar
 Snomen
  • Posts: 35
  • Joined: Sep 30, 2021
|
#94346
Hey, can anyone please explain why A is incorrect...I answered this question correctly but the diagram of A confuses me. I diagramed it: /GLS--->/PWP. My reasoning is if the author would provide this one as one of the premises.., would it make the conclusion more sound? It's a contrapositive of the conclusion.
Thank you in advance!
 Rachael Wilkenfeld
PowerScore Staff
  • PowerScore Staff
  • Posts: 1358
  • Joined: Dec 15, 2011
|
#94549
Hi snoman,

There are a few issues with the reasoning here. First, the conclusion you saw is right on pwp :arrow: gls, which matches what you saw gls :arrow: pwp. The problem though is that we aren't looking for something that makes the conclusion MORE sound. We are looking for the answer choice that describes why the conclusion doesn't follow.

You did a good job at identifying the issue. Our premise was the reverse of our conclusion, so we still needed a premise to support the relationship given in the conclusion. We showed already that a good legal system requires a well-paid police force, The argument just failed to show the reverse was true.

Answer choice (A) doesn't describe that. First, it uses the term "many." which means more than one, but is otherwise a vague term dependent on context. Many people is different than many Americans which is different than many European countries. Conditionality is a more specific and exact relationship. We are looking for the specific relationship you identified early on---it fails to establish that pwp :arrow: gls. That's answer choice (D).

Hope that helps!

Get the most out of your LSAT Prep Plus subscription.

Analyze and track your performance with our Testing and Analytics Package.